Showing that the range of a linear operator is not necessarily closed

In summary, we need to show that the range of the operator T is not closed. To do this, we can find a sequence of x_{n} \rightarrow x such that T(x_{n}) is in \ell^{2} but T(x) is not. Taking x_{n}=(1, \frac{1}{\sqrt{2}}, \frac{1}{\sqrt{3}}, ..., \frac{1}{\sqrt{n}}, 0, 0, 0 ,0,...) and T(x_{n})=(1, \frac{1}{2\sqrt{2}}, \frac{1}{3\sqrt{3}}, ..., \frac{1}{n\sqrt{n}},
  • #1
SP90
23
0

Homework Statement



Let [itex]T: \ell^{2} \rightarrow \ell[/itex] be defined by

[itex]T(x)=x_{1},\frac{1}{2}x_{2},\frac{1}{3}x_{3},\frac{1}{4}x_{4},...}

Show that the range of T is not closed

The Attempt at a Solution



I figure that I need to find some sequence of [itex]x_{n} \rightarrow x[/itex] such that [itex]T(x_{n})[/itex] is in [itex]\ell^{2}[/itex] but [itex]T(x)[/itex] is not.

I figured that if [itex]x=(\sqrt{1}, \sqrt{2}, \sqrt{3},...)[/itex] then [itex]T(x)[/itex] is the harmonic series, which is not square summable. The problem is, I can't think of any square summable [itex]x_{n}[/itex] which converge to that [itex]x[/itex].

I have a feeling that I'm taking the wrong approach to this. The thing is, I think the operator [itex]T(x)[/itex] must be bounded since [itex]k_{i}^{2} \geq \frac{1}{i^{2}}k_{i}^2[/itex]. Maybe I should be looking for some [itex]x_{n} \rightarrow x[/itex] such that [itex]T(x_{n})[/itex] converges to something other than [itex]T(x)[/itex].
 
Physics news on Phys.org
  • #2
I thought if I took [itex]x_{n}=(\frac{n}{1},\frac{n}{2^{2}},\frac{n}{3^{2}}...)[/itex], that would work, since [itex]x_{n} \rightarrow \frac{n\pi^{2}}{6}[/itex]. It's clear that [itex]T(x_{n})[/itex] exists for all n, but [itex]x_{n}[/itex] itself diverges. But then obviously the definition of the range being closed is that when [itex]T(x_{n})[/itex] has a limit, that limit is in the range. Obviously since [itex]T(x_{n})[/itex] diverges there's no criteria broken by this.
 
  • #3
I've solved this now, no need for a response, but if anyone ever comes across a similar problem here is the solution:

Take [itex]x_{n}=(1, \frac{1}{\sqrt{2}}, \frac{1}{\sqrt{3}}, ..., \frac{1}{\sqrt{n}}, 0, 0, 0 ,0,...)[/itex]

Obviously, as a series with a finite number of non-zero terms, it is square summable. Likewise [itex]T(x_{n})[/itex] also exists and is the sequence [itex]x_{n}=(1, \frac{1}{2\sqrt{2}}, \frac{1}{3\sqrt{3}}, ..., \frac{1}{n\sqrt{n}}, 0, 0, 0 ,0,...)[/itex]

Then [itex]T(x_{n}) \rightarrow y[/itex] where y is the infinite sequence of these. When each term is squared and the sum taken, we get the sum of reciprocal cubes. This is bounded by the sum of reciprocal squares, so it is square summable, so the sequence [itex]T(x_{n})[/itex] is convergent. But [itex]T^{-1}(y)[/itex] is the infinite sequence of reciprocal square roots. When each term is squared, the sum of these is the harmonic series, which is not convergent. So [itex]y[/itex] doesn't belong to the range of [itex]T[/itex] (since there is no [itex]x[/itex] in [itex] \ell^{2}[/itex] such that [itex]T(x)=y[/itex])
 

1. What is a linear operator?

A linear operator is a mathematical function that maps one vector space to another vector space while preserving the algebraic structure of the space.

2. How can the range of a linear operator be determined?

The range of a linear operator can be determined by applying the operator to all possible vectors in the domain and observing the resulting vectors in the range. The set of all resulting vectors is known as the range of the linear operator.

3. What does it mean for the range of a linear operator to be closed?

A closed range means that the range of the linear operator contains all possible limit points of the vectors in the range. In other words, every sequence of vectors in the range converges to a vector that is also in the range.

4. How can it be shown that the range of a linear operator is not necessarily closed?

To show that the range of a linear operator is not closed, one can provide a counterexample where there exists a sequence of vectors in the range that does not converge to a vector in the range. This would demonstrate that the range is not closed.

5. What implications does a non-closed range have for a linear operator?

A non-closed range can have significant implications for a linear operator. It means that the operator does not preserve all limit points and may not have a well-defined inverse. This can affect the stability and uniqueness of solutions to linear equations involving the operator.

Similar threads

  • Calculus and Beyond Homework Help
Replies
1
Views
255
Replies
1
Views
571
  • Calculus and Beyond Homework Help
Replies
4
Views
882
  • Calculus and Beyond Homework Help
Replies
9
Views
971
  • Calculus and Beyond Homework Help
Replies
2
Views
833
  • Calculus and Beyond Homework Help
Replies
4
Views
306
Replies
2
Views
526
  • Quantum Physics
Replies
31
Views
2K
  • Calculus and Beyond Homework Help
Replies
17
Views
614
  • Calculus and Beyond Homework Help
Replies
4
Views
1K
Back
Top